11. Factor for

12 – 8.1 + 16​

Answers

Answer 1
I’m going to go ahead and say the answer is 19.9... sorry if it’s wrong.
Answer 2

Answer:

2(6-4.05+8)

Step-by-step explanation:

2(6-48+43.05+8)


Related Questions

Given the function d (t) = 50t, the variable t represents which of the following
Output
Independent variable
Function
Input
Depended variable

Answers

Answer:

The input of the function

Step-by-step explanation:

Megan had $235.17 in her bank account. She withdrew $77.98 from the account to spend
on clothing. How much money does she have left in her bank account now?

Answers

Answer: 157.19

Step-by-step explanation:

157.19 that would be the answer

Salena needs to purchase 9 skirts for a new job. her friend, Jamie goes with her each of the skirts costs $41. Salena says that she can find the total cost by multiplying 9 and 4 and adding that to product of 9 and 1. Jamie disagrees and says that she needs to find the product of 9 and 40 and add that to the product of 9 and 1. Who is correct and why?

Answers

Answer:

Jamie is correct. This is because 41 is a combination of 40 and 1. Salena thinks that the 4 is just a 4, however, the 4 is in the tens place, so the 41 without the 1 is 40. The answer to this 41 times 9 is 369. Not 45.

Step-by-step explanation:

Each skirt is $41

Salena needs 9

(9*40) + (9*1)

360   +   9

    $369

what is 10x plus 50 - 2 divided by 6 + 100 times 8 - -2.

Answers

Answer: the answer is -947 hope this helped.

Step-by-step explanation:

Can someone please help me I need help and I can’t understand this plsssss

Answers

1a. Multiply a variable by 13, add 2.

1b. Subtract two from both sides, divide both sides by 13. X=3

2a. Subtract 3 from a variable; divide by 5

2a. Multiply both sides by 5,add 3 to each side. x=-2
Solution (1) :

Solution (a) :

step 1. Multiply the variable x with 13 .

step 2. Add 2 to it .

Solution (b) :

Step 1 : Subtract 2 from both sides : 13x + 2 - 2 = 41 - 2 = 39

Step 2 : Divide both the sides by 13 : 13x ÷ 13 = 39 ÷ 13 = 3

x = 3

Solution (2) :

Solution (a) :

Step 1. Subtract 3 from the variable .

Step 2. Divide the result by 5 .

Solution (b) :

[tex]Step \: 1. \: add\: 3 \: to \: both \: the \: sides : \frac{x - 3}{5} + 3 = - 1 + 3

[/tex]

Step 2 : Multiply both the sides by 5 : 5 × 1/5x = 5 × -2/5

x = -2

ax+6=15 a is a negative, what must be true about x?

Answers

Answer:

x= 9/a

Step-by-step explanation:

Please help me with all the answers please say like 1: the answer 2: the answer like that thank you so much

Answers

1: 8200,82000,820000 yen
2:12
3:8
4:6
5:4 months

Solve for x for the following pls!
1) 7+3x+1=4x+5
2) 8+2x+18=x+19
3) 6+2x+13=x+26
would appreciate it sm

Answers

Answer:

1.x=3

2. x= -7    

3. x=7

Step-by-step explanation:

subtract 8 from the input to get the output
divided the input by 3 to get the output

Answers

Answer: that will be 5

Step-by-step explanation:

Answer:

6

Step-by-step explanation:

Pls I need help ASAP

Answers

Answer:

the answer is 1/8

thank you

if you loved my answer then please like it and mark as brainliest

The toco toucan, the largest member of the toucan family, possesses the largest beak relative to body size of all birds. This exaggerated feature has received various interpretations, such as being a refined adaptation for feeding. However, the large surface area may also be an important mechanism for radiating heat (and hence cooling the bird) as outdoor temperature increases. Here are data for beak heat loss, as a percent of total body heat loss, at various temperatures in degrees Celsius:

Temperature (degrees Celsius) 15 16 17 18 19 20 21 22 23 24 25 26 27 28 29 30
Percent heat loss from beak 35 36 38 28 41 43 55 46 39 54 45 58 60 56 62 67
a. The equation of the least-squares regression line for predicting beak heat loss, as a percent of total body heat loss from all sources, from temperature is: _____ (Use decimal notation. Give your answer to four decimal places.)

b. Use the equation to predict (
±
0.01) beak heat loss, as a percent of total body heat loss from all sources, at a temperature of 25 degrees Celsius.

c. What percent (
±
0.01) of the variation in beak heat loss is explained by the straight-line relationship with temperature?

d. Find the correlation r (
±
0.001) between beak heat loss and temperature.

Answers

Answer:

Kindly check explanation

Step-by-step explanation:

Given the data:

Temperature (degrees Celsius) : 15 16 17 18 19 20 21 22 23 24 25 26 27 28 29 30

Percent heat loss from beak : 35 36 38 28 41 43 55 46 39 54 45 58 60 56 62 67

Using an online regression calculator ; the regression equation obtained is :

ŷ = 2.0927X + 0.6029

X = independent variable

Y = predicted variable

2.0927 = slope

0.6029 = intercept

B.) temperature = 25

ŷ = 2.0927(25) + 0.6029

= 52.9204

C.) the explained variance is the value of the coefficient of determination (R²) which is the square of the correlation Coefficient

0.8785² = 0.7718

D.) the correlation Coefficient r is 0.8785 using the Coefficient of regression calculator

I don't understand with this question, can you help me with this pls?

Answers

Answer:

By moving the decimal at the end of the number to the correct amount of places for x 10 you move it once to the right for x 100 you move it right twice.

Step-by-step explanation:

Show that each conjecture is false by finding a counterexample.


1.For any integer n, n3 > 0.


2. Each angle in a right triangle has a different measure.


3.For many years in the U.S, each bank printed its own currency. The variety of different bills led to widespread counterfeiting. By the time of the Civil War, a significant fraction of the currency in circulation was counterfeit. If one soldier had 48 bills, 16 of which were counterfeit, and another soldier had 39 bills, 13 of which were counterfeit, make a conjecture about what fraction of bills were counterfeit at the time of the Civil War.

Answers

Answer:

1.   [tex]n: n^3 > 0[/tex] is not true for negative integers

2.  [tex]45\ and\ 45[/tex] show that all angles do not have different measure

3.  [tex]Fraction = \frac{1}{3}[/tex]

Step-by-step explanation:

Solving (1):

[tex]n: n^3 > 0[/tex]

Take

[tex]n = -1[/tex]

[tex]-1^3 = -1[/tex]

[tex]n: n^3 > 0[/tex] is not true for negative integers

Solving (2):

The angles of a right angled triangle are:

[tex]45, 45\ and\ 90[/tex]

[tex]45\ and\ 45[/tex] show that all angles do not have different measure

Solving (3):

When

[tex]Bills = 48[/tex]

[tex]Counterfeit = 16[/tex]

The fraction is calculated as thus:

[tex]Fraction = \frac{Counterfeit}{Bills}[/tex]

[tex]Fraction = \frac{16}{48}[/tex]

[tex]Fraction = \frac{1}{3}[/tex]

Similarly, when

[tex]Bills = 39[/tex]

[tex]Counterfeit = 13[/tex]

[tex]Fraction = \frac{Counterfeit}{Bills}[/tex]

[tex]Fraction = \frac{13}{39}[/tex]

[tex]Fraction = \frac{1}{3}[/tex]

x+12x-13 = 0 and azo, what
is the value x+lo?​

Answers

Wat this doesn’t make sense

Consider the hexagon ABCDEF what is the value of X? What is the measure of angle A? what is the measure of angle BCG?

Answers

Answer:

x = 52.5

<A = 152.5°

<BCG = 45°

Step-by-step Explanation:

Sum of interior angles of n-sided polygon = (n - 2)180

Sum of interior angles of hexagon = (6 - 2)180 = 4(180) = 720°

Therefore, sum of all the interior angles in the given hexagon = 720.

Thus:

3x - 5 + 2x + 140 + 135 + 2x + 30 + x = 720

Solve for x. Add like terms.

8x + 300 = 720

Subtract 300 from both sides

8x = 720 - 300

8x = 420

Divide both sides by 8

8x/8 = 420/8

x = 52.5

<A = (3x - 5)

Plug in the value of x

<A = 3(52.5) - 5 = 152.5°

<BCG = 180 - (2x + 30) (angles on a straight line)

Plug in the value of x

<BCG = 180 - (2(52.5) + 30)

<BCG = 180 - (105 + 30)

<BCG = 180 - 135

<BCG = 45°

whats 62.7×2.45......?

Answers

Answer:

153.615

Step-by-step explanation:

62.7 × 2.45=153.615

Tell whether the sum is positive, negative, or zero without adding them.
-8 + 20

Answers

Answer:

The sum is positive.

Step-by-step explanation:

The 20 is a far greater value than -8, so no matter what, when you add them the sum will be positive. Though this would not be the same were the -8 greater than -20.

Hope this helps!

BI
Challenge 2
A: Can you find the
area of the circle?
r=x +1
B: Can you find the
circumference of
the circle?

Answers

Answer:

Area of the circle = pi * r^2 = pi * (x+1) ^ 2

Circumference of the circle = 2* pi * r = 2*pi*(x+1)

CONSTRUCTED-RESPONSE ITEM 14. Four people each deliver food to people's homes.
19
Constructed Responses
..
Curtis charges a flat fee of $2.50 for each delivery plus $0.20 per mile for each mile he drives. For one delivery, Curtis drives 6 miles.
...
A. How much does Curtis charge to deliver the food?

Answers

Answer:

I'm Pretty Sure it is $16.20

Step-by-step explanation:

First, you add 2.50 + 0.20 and you get 2.70. Then, you get 2.70 x 6 miles and you get 16.20. Hope this helps! :)

Which of the following numbers is irrational?

Answers

The second one I think

Cliff needs 6 meters of wood for a building project. He already has 3
meters of wood.
Approximately how many feet of wood does Cliff need? (One meter
is approximately 3.3 feet.)
1 foot
3 feet
6 feet
10 feet

Answers

letter B 3 feet

Step-by-step explanation:

he needs three more to get 6 meters of wood

10, because he needs 3 more meters and 3 meters = 9.9 feet

guys what is 265 divided by 53

Answers

Answer:

5

Step-by-step explanation:

265/53= 5

Answer:

265÷53=5

Step-by-step explanation:

just get a calculator

diride 660cm in the ratio of 5:6​

Answers

Answer:

Thus, the measure 660 cm is divided into 300 + 360 in the ratio of 5:6

Step-by-step explanation:

Equations

We are required to divide 660 cm in the ratio 5:6, or 5/6.

Let's call:

x = first part of 660

660 - x = second part of 660

The ratio between them is 5/6.

[tex]\displaystyle \frac{x}{660-x}=\frac{5}{6}[/tex]

The above equation will result in x as the smaller part of 660.

Multiply by (660-x)*6 to eliminate denominators:

[tex]\displaystyle (660-x)*6\cdot\frac{x}{660-x}=(660-x)*6\cdot\frac{5}{6}[/tex]

Operating:

[tex]6x=(660-x)*5[/tex]

[tex]6x=3300-5x[/tex]

Simplifying:

[tex]6x+5x=3300[/tex]

[tex]11x=3300[/tex]

Solving:

[tex]\displaystyle x=\frac{3300}{11}=300[/tex]

The other part is 660 - 300 = 360

Thus, the measure 660 cm is divided into 300 + 360 in the ratio of 5:6

How do you find what p is?

Answers

Answer:

p > -3

Step-by-step explanation:

This is an alternating p-series.  It is absolutely convergent if│an│converges.  1 / nᵖ⁺³ converges if the exponent is greater than 1.

p + 3 > 1

p > -2

The series is conditionally convergent if │an│diverges.  1 / nᵖ⁺³ diverges if the exponent is greater than 0 and less than or equal to 1.

0 < p + 3 ≤ 1

-3 < p ≤ -2

Combining these two, we get p > -3.

in quadrilateral abcd below,

Answers

Answer: where is the quadrilateral

Step-by-step explanation:

Quadrilateral GHJK has vertices G(2, 3), H(8, 2), J(6, 8), and K(3, 6). It is transformed according to the rule T(–4, –5).

What are the coordinates of G”?

(–7, 3)
(–2, 2)
(–1, –7)
(2, –2)

Answers

Answer:

the transformation T(a, b) can be change the pre image to the final image by following the rule as we explain below

if A(x, y) is a pre image, then A ' ( x', y') is its image, and x' = a+x, y' = b + y, where because of T(a, b) transformation

in our case Quadrilateral GHJK has vertices G(2, 3), so the coordinates of G”

can be found with

T(-4, -5) applied to G(2, 3) ⇒ G' (-4 +2, -5+3)=G' (-2, -2)

T(-4, -5) applied to G' (-2, -2) ⇒ G"(-4-2, -5-2) =G"(-6, -7)

the coordinates of G” are (-6, -7)

Step-by-step explanation:

i hope it helps?

Answer:

its c on edg (-1, -7)

Step-by-step explanation:

(-8) X 2/3 i NEED the answer help me

Answers

-5.33333333333

I used a calulator can I get brainliest please

Evaluate 8 ÷ -2 · 4²+9. Thanks! Will give brainiest if possible

Answers

the answer is -55 , because you have to simply the 14^2 to 16
For more anwsers for questions like these use math away love even thought the ads might be irritating it’s a great source ✨

6. The store paid $2.50 for a book and sold it for $5.25. What is the profit as a percent?

Answers

7.75 I hope this helps

What’s the difference between m=change in y and change in x and m =y2-y1and x2-x1

Answers

it's the same thing

[tex]slope = \frac{Δy}{Δx} or \frac{change \: in \: y}{change \: in \: x} = \frac{y2 - y1}{x2 - x1} = \frac{rise}{run} [/tex]

Other Questions
Pham can work as many or as few hours as she wants at the college bookstore for $12 per hour. But due to her hectic schedule, she has just 15 hours per week that she can spend working at either the bookstore or other potential jobs. One potential job, at a caf, will pay her $15 per hour for up to 6 hours per week. She has another job offer at a garage that will pay her $13 an hour for up to 5 hours per week. And she has a potential job at a daycare center that will pay her $11.50 per hour for as many hours as she can work.If her goal is to maximize the amount of money she can make each week, how many hours will she work at the bookstore? What happens when the elements in group 2 react with water? Where does a carnivore directly get its energy from? Can someone plz plz help me with the last 2 problems #5 and #6 how can you tell that the yeti crabs are thriving? ANSWER MY QUESTION FIRSTCan you help me with these questions, please? Jack bought 5.94 pounds of beef for dinner. What is the first step in rounding this number to the nearest tenth? What is the next step? What is 5.94 rounded to the nearest tenth? The harappan civilization was known for HELP PLEASE HS GEOMETRY 50 POINTS!!!!! hey everyone . sorry . I always answered your all questions from Google which was a cheating . sorry if u think that I am a cheater. but now I will try to answer by my self Ramone adds a mixture of salt, sugar, and raisins to a bowl of water. He notices that after stirring and waiting for everything to settle, his new mixture has two visible parts: a clear liquid with raisins. What does this tell you about his new mixture? (1 point) aThe mixture is a new substance with different properties. bThe salt evaporated out of the mixture into the air. cThe salt and sugar dissolved in the water. dThe water made some of the ingredients disappear.Helppp I mark as brainliest If you could, please check my dialogue. I'm not too sure if it is all grammatically correct. Can anyone help me do my missing assignments? Im trying but I just cant handle it :(? Read the text "New Zealand PM Jacinda Ardern earns praise at home and abroad". Use the text to answer questions 1-7.1. Select the best objective summary for the tant (1.12)A Jacinda Ardern is both criticized and loved for her polical views and how she governs hercountryB. Across the world, Jacinda Ardem is admired for her progressive politics and ability to be aNew Zealand PM Jacinda Ardern earnssympathetic leader to her countrypraise at home and abroadC Jacinda Ardem is expanding people's ideas of what a world leader should be by being awoman mother and Muslimin eD. Throughout New Zealand, people are expressing their concern for Muslims in theircommunities after the terrorist attacks What requirements must be met to obtain a marriage license?,Only one person applying for marriage must be present.Each person must be 21 or olderOcEach person must present proper identification, such as a driver's licenseOD. Each person may be married to someone else An aircraft factory manufactures airplane engines. The unit cost (the cost in dollars to make each airplane engine) depends on the number of engines made. If engines are made, then the unit cost is given by the function . What is the minimum unit cost?C(x)=x^2-520x+73458 can someone PLEASE help me on this PLEASE.The given table of values represents a linear equation. What is the slope? A stack of books is 62 inches high. How high is it in feet and inches? Write each equation/inequality in slope-intercept form.y- 5 = 1/3 (x-6)